No entiendo cómo concluyes la última desigualdad en la línea que empieza Introduciendo algunas cifras aleatorias en wolframalpha se obtiene pero que parece contradecir lo que has escrito cuando
Esto sustituye a mi respuesta original, que era defectuosa pero llevaba en la dirección correcta (creo).
Dejaré que la serie comience en algún La razón de ello se verá más adelante.
Ampliar en (con para ):
Intercambiemos los sumandos y preocupémonos después de si estuvo bien:
Ahora la suma interna converge según Prueba de Dirichlet . De hecho podemos proceder como en la demostración de la prueba de Dirichlet para acotar su límite (suprimiendo los índices mientras trabajamos en fijo ):
con utilizando la fórmula de reducción de potencia para el coseno y absorbiendo el factor en los cosenos sumando a su frecuencia.
Así, podemos aplicar la prueba comparativa con las siguientes series:
Esto demuestra por qué necesitamos empezar en algún en lugar de lo que no funcionaría ya que .
Ahora todo depende del comportamiento de . El coseno se acerca arbitrariamente a ; cuando entra en de un múltiplo impar de entonces . Heurísticamente hablando, podríamos esperar que esto ocurriera cada enteros, por lo que en promedio estos picos no destruirían la decadencia a largo plazo con (que puede hacerse cuantitativamente, aunque no cualitativamente, más fuerte aumentando ). Sin embargo, no sé cómo hacer que este argumento sea riguroso, o si eso es siquiera posible dada la "imprevisibilidad" de . Es interesante que esta prueba pueda depender de los detalles de de esta manera. [Actualización: Gracias a George Lowther por los comentarios que apuntan a esta página de MathWorld lo que da un límite superior a la medida de irracionalidad de (nunca lo había oído antes :-). Que esto sea finito implica que puede estar limitada por una potencia de y, por tanto, la suma exterior converge (uniformemente), creo que eso completa la demostración].
Suponiendo que se pueda resolver el problema de los picos, aún tenemos que volver a la intercambiabilidad de las sumas. Este teorema muestra que podemos intercambiar las sumas si la convergencia de
es uniforme en -- que lo es, ya que se eliminó en la derivación del límite .
En resumen, esta derivación muestra que se puede demostrar que la serie converge si los picos irregulares en que se producen porque se acerca arbitrariamente a impar múltiplos de puede demostrarse que no destruye la convergencia de la suma exterior sobre .
Gracias por esta interesante pregunta :-).
No entiendo cómo concluyes la última desigualdad en la línea que empieza Introduciendo algunas cifras aleatorias en wolframalpha se obtiene pero que parece contradecir lo que has escrito cuando
@Derek Jennings: Lo miraré más tarde esta noche; puede que se me haya escapado un factor 2 en alguna parte, pero la idea general es escribir el coseno como una suma de exponenciales y usar la fórmula para la suma parcial de una serie geométrica, así que creo que la desigualdad es correcta salvo quizás por un factor erróneo. Más adelante...
@Derek Jennings: Tienes razón, se me ha escapado un factor 2. Gracias por darte cuenta. Si reduces las dos fracciones que obtienes de la serie geométrica a un denominador común, obtienes 8 términos de valor absoluto en el numerador, y en el numerador, por lo que el límite tiene que ser .
Insertar \sum_{n=2}^{infty} (-1)^n/(ln(n)+cos(n))
en el cuadro de búsqueda de Wolfram Alpha da un valor aproximado de -4,552162944044021. Por increíble que sea la suma real, puedes estar un poco seguro de su convergencia.
I-Ciencias es una comunidad de estudiantes y amantes de la ciencia en la que puedes resolver tus problemas y dudas.
Puedes consultar las preguntas de otros usuarios, hacer tus propias preguntas o resolver las de los demás.
8 votos
Esto se mencionó como ejemplo de una serie para la que se necesita saber bastante sobre aproximaciones racionales a π para demostrar la convergencia, en un comentario a la siguiente pregunta MO. mathoverflow.net/questions/54758/
0 votos
@George, gracias por tu comentario. Me encantaría conocer los detalles si alguien aquí los conoce....
0 votos
No he visto una prueba de esto, pero yo piense en el resultado se obtendrá a partir de la equidistribución, mostrando que los términos positivos y negativos de la serie se anulan en gran medida entre sí. Probablemente tendrás que utilizar el hecho de que π tiene una medida de irracionalidad finita para obtener un límite lo suficientemente bueno como para demostrar la convergencia, como con la respuesta de David Speyer a esta pregunta anterior: math.stackexchange.com/questions/2270/
1 votos
Si f es una función suave con periodo 1 y α tiene medida de irracionalidad finita, entonces se puede acotar ∑Nn=1f(αn)−N∫10f(x)dx . A continuación, puede vincular ∑Nn=1(−1)nf(αn) . Creo que esto podría funcionar para esta suma aplicando equidistribución al cosn parte, pero aún no he revisado los detalles.
1 votos
Existe otra serie cuya convergencia depende de sutiles detalles de π es decir, cuán raramente un seno o un coseno es "extremo" cuando se evalúa en valores enteros, y que es el Serie Flint Hills ∞∑n=11n3(sinn)2 llamado así por el topónimo por C. A. Pickover .